LSAT and Law School Admissions Forum

Get expert LSAT preparation and law school admissions advice from PowerScore Test Preparation.

 ovibalaj
  • Posts: 8
  • Joined: Jun 22, 2015
|
#18981
"Logical Reasoning - Question Type Training" Chapter 2: Must Be True; Question 3; page 21.

Stimulus:

"A population of game ducks at a western lake contains 55 males to every 45 females..."

Question Stem:

"Which one of the following can be inferred from the passage?"

** Inferred in LSAT = must be true


Analysis [Primary Objectives]:

* P.O 1. Determine whether the stimulus contains an argument or if it is only a set of factual statements.

"A population of game ducks at a western lake contains 55 males to every 45 females...."


Because – premise indicator
We can infer – not in the conclusion indicator table, but I think it translates to "It must be true that". Is this correct?


This stimulus is an Argument.

* P.O 2. If it is an argument, identify the conclusion of the argument. If it is a fact set, examine each fact.

This stimulus is an Argument. It's main conclusion is:

This is preceded by the statement – "We Can Infer That"

"…we can infer that the greater the disparity in overall sex ratios, the greater the percentage of older male ducks in the population."


* P.O 3. If the stimulus contain an argument, determine if the argument is strong or weak.

The argument is strong. It is a direct relationship. Is this a good enough reason?

* P.O 4. Read closely and know precisely what the author said. Do Not Generalize.

It seems that it is summed up in the conclusion.

* P.O 5. Carefully read and identify the question stem. Do not assume that certain words are automatically associated with certain types.

"Which one of the following can be inferred from the passage?"

It is asking which one of the answers must be true.


* P.O 6. After reading the question stem, take a moment to mentally formulate your answer to the question stem. [PREPHRASE]

What should I do here?


* P.O 7. Always read each five of the answer choices.

Done…

* P.O 8. Separate the answer choices into Contenders and Losers. After you complete the process, review the contenders and decide which answer is the correct one.

Stimulus:

"A population of game ducks at a western lake contains 55 males to every 45 females..."

Question Stem:

"Which one of the following can be inferred from the passage?"

Relationship from conclusion:

Larger the disparity between males and females → larger the number of older male ducks


Answer Choices:

A. The population of game ducks at the western lake contain a lower percentage of adult males than the population at the eastern lake contains.

CONTENDER
→ it reflects the relationship. Western lake (55 males to 45 females, n=10). Eastern Lake (65 Males to 35 Females, n=30). Eastern Lake has more older male ducks than Western Lake.
This passes the Fact Test and is in direct connection with the information in the stimulus.

B. The population of game ducks at the eastern lake contains a higher percentage of nonadult game ducks than the population at the western lake contains.

LOSER
→ it goes against the relationship. Western lake (55 males to 45 females, n=10). Eastern Lake (65 Males to 35 Females, n=30). Eastern Lake has more older male ducks than Western Lake.

C. The total number of male game ducks is higher in the eastern lake's population than in the western lake's population.

CONTENDER
→ this information is true. Why is this answer wrong?

D. The number of nonadult ducks hatched in a breeding season is higher in the eastern lake's population than in the western lake's population.

LOSER
→ I don't think this can be determined from the stimulus. Is that so?

E. Adult female game ducks outnumber non adult female game ducks in the eastern lake's population.

LOSER
→ I don't think this can be determined from the stimulus. Is that so?
 Nikki Siclunov
PowerScore Staff
  • PowerScore Staff
  • Posts: 1362
  • Joined: Aug 02, 2011
|
#18991
Hi Ovi,

Thanks for your question. Just a reminder to please remove the copyrighted content from your post.

Your analysis is correct: this is an argument ("it can be inferred that..." serves as a conclusion indicator), and it's a pretty solid one. As I mentioned earlier, evaluating whether an argument is valid or not is only relevant to questions in the Second and Third Families (Help/Hurt), because Prove-type questions require us to assume that the information in the stimulus is valid and true.

As far as a prephrase is concerned, well - the conclusion is already stated in the stimulus, so let's see if there is anything else you can prephrase given the facts (and the conclusion, which you should assume to be true): Since the m/f ratio in the eastern lake is higher than that in the western lake, and greater disparity in sex ratios means a greater percentage of older male ducks, we can prephrase that that there is a higher proportion of old male ducks in the eastern lake than in the western lake. This prephrase agrees with answer choice (A).

Note, however, that we know nothing about the NUMBER of ducks in either lake: this is all about m/f ratios, not about the quantity of ducks that reside there. This is why answer choice (C) is wrong: there could still be more male ducks in the western lake than in the eastern lake, even if the m/f ratio is smaller in the western lake, provided there are just a lot more ducks in the western lake. For instance:

Western lake: 1000 ducks: 550 males, 450 females
Eastern lake: 100 ducks: 65 males, 34 females

Answer choice (C) would only be true if we assumed that the eastern lake has at least as many ducks as the western lake. Does that make sense?

The remaining answer choices state comparisons that cannot be deduced given the information contained in the stimulus. They can easily be eliminated because they deal with numbers, not percentages. Absent any information about the total number of ducks in either lake, no numerical inference can be drawn from the information provided.

Hope this helps! :)
 Sherry001
  • Posts: 81
  • Joined: Aug 18, 2014
|
#21367
Hello ;
I got this question right but I had trouble eliminating C. Could you please see if my reasoning is correct. Numbers and percentages always throw me off.



1- population of game ducks in western 55 males:45 fem
2- population of game ducks in eastern 65 males to 35 fem
3- among the ducks not yet bred in eastern there are only slightly more males then females.
4- but among the olde ducks the number of males greatly exceeds number of females.

C: we can infer the grater the disparity in overall sex ratios the grater the percentages of older make ducks in the population.

A) I chose this choice over c because I learned from my bibles that in must be true questions to be careful not to take percentages to indicate any raw numbers.

C) there are two reasons I didn't choose this. 1) because it has actual number instead of percentage and 2) because I thought since the stimulus only gave us info about ducks "not yet bred in eastern lake", it could be possible that the ducks to be bred in the western lake be equal to the males in eastern" ( and still exceed the number of females ).

Thanks !
Sherry
 Lucas Moreau
PowerScore Staff
  • PowerScore Staff
  • Posts: 216
  • Joined: Dec 13, 2012
|
#21368
Hello, Sherry,

I'm glad you got this question right - this is a tough question to appear so early in the test! :)

You are correct in your reasoning for rejecting answer choice C, mainly the part about the numbers versus the percentages. We know what the ratios of male to female ducks are for the western and eastern lakes, but not the total number of ducks. Maybe the eastern lake has 100 ducks and the western lake has 10,000. As such, we can't say C must be true.

Hope that helps,
Lucas Moreau
 MannyH
  • Posts: 10
  • Joined: Jun 19, 2018
|
#49279
Hello,

The question I have is a concern of clarity. Due to the fact that MBT questions do no contain a conclusion, can we say that the "conclusion" in this stimulus is actually a single conditional statement?

M>F --> Greater percentage of overall male ducks
 Adam Tyson
PowerScore Staff
  • PowerScore Staff
  • Posts: 5153
  • Joined: Apr 14, 2011
|
#49333
The way you diagrammed that, Manny, is a tautology - if there are more males than females, there are more males than females. If you meant to say "adult" or "older" male ducks, rather than "overall," then you've got it in a nutshell.

This is a rare MBT question in that it does actually have a conclusion, and it's the one you identified. Our task is to use that conclusion as an intermediate conclusion, and to draw an additional inference based on that conclusion and the other info we got about the populations in the two lakes. That info works together to prove answer A for us.

Most of the time, you're right that MBT questions contain no conclusion, and this fairly ancient question is the rare exception today. However, even if you find a conclusion, you could still be asked to draw another one based upon it.
User avatar
 ericsilvagomez
  • Posts: 49
  • Joined: Oct 16, 2023
|
#104211
Hi,

Is there an explanation for this problem that also explains the other answer choices, like in the other explanations?
 Luke Haqq
PowerScore Staff
  • PowerScore Staff
  • Posts: 742
  • Joined: Apr 26, 2012
|
#104222
Hi ericsilvagomez!

We will work on getting a full explanation for this one. In the meantime, are there particular answer choices that gave you difficulties? If you selected the incorrect answer choice but aren't sure why it's wrong, or even if there are a couple of answer choices you were unsure about, just let us know. If you can explain your reasoning as to why you aren't persuaded by the correct answer, or why you like a different answer, we will be happy to address that.
User avatar
 ericsilvagomez
  • Posts: 49
  • Joined: Oct 16, 2023
|
#104229
Hi,

I selected answer choice C and am unsure why A is correct. However, my answer was also a guess. I chose C because I thought the disparity in the sex ratio of 65 males for every 35 females meant the total number of male ducks was higher in the Eastern lake than in the Western lake. Is answer choice A correct because there is less disparity in the sex ratio?
 Luke Haqq
PowerScore Staff
  • PowerScore Staff
  • Posts: 742
  • Joined: Apr 26, 2012
|
#104239
Hi ericsilvagomez!

Happy to address answer choices (C) and (A). Regarding (C), note that we're not told anything about the actual number of game ducks. Rather we're told that there are "55 males to every 45 females" in the western lake, and "65 males for every 35 females" in the eastern lake. So we're given information in terms of proportions rather than actual numbers. This is why (C) is incorrect--we don't know that the actual number in one lake is greater than the other lake.

What we do know, however, is that there is a greater disparity in overall sex ratios in the eastern lake than in the western lake. We can combine that with what we're told in the final sentence: "we can infer that the greater the disparity in overall sex ratios, the greater the percentage of older male ducks in the population." From this we can infer that the eastern lake contains a higher percentage of adult males than the western lake--or in the language of answer choice (A), "the western lake contains a lower percentage of adult males than the population at the eastern lake."

Get the most out of your LSAT Prep Plus subscription.

Analyze and track your performance with our Testing and Analytics Package.